Thomas' Calculus 13th Edition

Published by Pearson
ISBN 10: 0-32187-896-5
ISBN 13: 978-0-32187-896-0

Chapter 10: Infinite Sequences and Series - Section 10.1 - Sequences - Exercises 10.1 - Page 571: 111

Answer

The sequence $a_n$ is monotonic and bounded.

Work Step by Step

Consider $a_n=\dfrac{3n+1}{n+1}$ for all $n \in N$ Also, $a_{n+1}=\dfrac{3(n+1)+1}{(n+1)+1}=\dfrac{3n+4}{n+2}$ for all $n \in N$ we can see that $\dfrac{3n+1}{n+1} \lt \dfrac{3n+4}{n+2}$ for all $n \in N$ This means that $a_n \lt a_{n+1}$ for all $n \in N$ .Thus $a_n$ increases and is monotonic. Now, to check whether the sequence is bounded or not we will consider $0 \lt \dfrac{3n+1}{n+1}=2(\dfrac{n}{n+1})+1$ Also, $\dfrac{n}{n+1} \lt 1$ for all $n \in N$ This implies $0 \lt (2)(1) +1=3$ for all $n \in N$ so, $a_n$ is bounded. Therefore, we can see that the sequence $a_n$ is monotonic and bounded.
Update this answer!

You can help us out by revising, improving and updating this answer.

Update this answer

After you claim an answer you’ll have 24 hours to send in a draft. An editor will review the submission and either publish your submission or provide feedback.